PT55.S1.Q15 - zack's coffeehouse

E.T.90066-1E.T.90066-1 Alum Member
edited December 2015 in Logical Reasoning 377 karma
http://7sage.com/lsat_explanations/lsat-55-section-1-question-15/
I cannot understand why Answer E) is wrong! Can someone please look at this question, its rather tricky.

If almost every = (51-99) why can I not conclude that on some W Zack does not offer half priced coffee all day?

Comments

  • retaker2014retaker2014 Alum Member
    edited April 2014 99 karma
    Hey there, I'll give this a shot.

    Going back to the stimulus:

    Premise 1: Free Poetry readings take place almost every Wednesday.
    Premise 2: If a poetry reading takes place then there is half priced coffee all day.

    According to choice E, there are some Wednesdays when there is no half-priced coffee.

    But quick, back to to the stimulus:

    If PR (which occurs almost every wednesday) ---> then HPC

    But according to Choice E:

    No HPC on some Wednesdays.

    What this choice is betting on, is you making the mistake of thinking that free poetry is necessary for half priced coffee to occur. This is not the case.

    The contrapositive of the stimulus is:

    No HPC ---> No Poetry Reading

    NOT

    HPC ---> Poetry Reading

    In summa:

    There cannot be a poetry reading on Wednesday if there is no half priced coffee that day. BUT, Wednesday could simply be half-priced-coffee-day, every week, whether or not there is a poetry reading.

    The 'almost every Wednesday' is there to trick you - it only refers to how often poetry occurs, not to how often half priced coffee occurs. Half priced coffee is necessary for poetry to occur but not sufficient. Therefore we have no idea, if there are some Wednesdays without half priced coffee. There could be but we don't know for sure, and so choice (E) is wrong.

    Hope this helps.
  • E.T.90066-1E.T.90066-1 Alum Member
    377 karma
    Well, is sort of helps.
    This is how I initially diagramed it.
    W-Most->FPR
    FRP->HPC
    Linked up: W-most->HPC

    I see, I cannot conclude from a most statement that "some not"....
  • retaker2014retaker2014 Alum Member
    edited April 2014 99 karma
    Yeah....you shouldn't have diagrammed W-Most -> FPR, or inferred what you did..
    That's where you got lost.
    The 'almost every wednesday' is there to tell you how often the FRP --> HPC relationship occurs. It does not produce any logic statements by itself.
Sign In or Register to comment.